¿Por qué una integral de acción debe ser estacionaria? ¿Sobre qué base estableció Hamilton este principio?

El principio de Hamilton establece que un sistema dinámico siempre sigue una trayectoria tal que su integral de acción es estacionaria (es decir, máxima o mínima).

¿Por qué la integral de acción debe ser estacionaria? ¿Sobre qué base estableció Hamilton este principio?

Cabe señalar que este es el "principio de Hamilton", es decir, no es exactamente lo mismo que la "mecánica hamiltoniana [clásica]" (es decir, donde está involucrado un hamiltoniano real) y eso no es nada específico sobre QM.
En las ecuaciones de Euler Lagrange. La condición necesaria para que L sea un punto extremo es que satisfaga las ecuaciones EL. Entonces, el principio de Hamilton no es en realidad un principio. Puedes pensar en QED, en QM,... pero es solo por una razón matemática.

Respuestas (7)

Las notas de la semana 1 del curso de mecánica lagrangiana de John Baez dan una idea de las motivaciones de los principios de acción.

La idea es que la mínima acción podría considerarse una extensión del principio del trabajo virtual. Cuando un objeto está en equilibrio, se necesita cero trabajo para hacer un pequeño desplazamiento arbitrario sobre él, es decir, el producto escalar de cualquier vector de desplazamiento pequeño y la fuerza es cero (en este caso porque la fuerza misma es cero).

Cuando un objeto está acelerando, si agregamos una "fuerza de inercia" igual a metro a , entonces un desplazamiento pequeño, arbitrario y dependiente del tiempo de la trayectoria verdadera del objeto volvería a tener un producto escalar cero con F metro a , la fuerza verdadera y la fuerza de inercia sumadas. Esto da

( F metro a ) d q ( t ) = 0

A partir de ahí, algunos cálculos que se encuentran en las notas conducen a la integral de acción estacionaria.

Baez habla más de D'Alembert que de Hamilton, pero de cualquier manera es una mirada interesante a los orígenes de la idea.

Tenga en cuenta que el principio del trabajo virtual se llama principio D'Alembert: en.wikipedia.org/wiki/D%27Alembert%27s_principle

También existe el enfoque de Feynman, es decir, la mínima acción es verdadera clásicamente solo porque es verdadera en la mecánica cuántica, y la física clásica se considera mejor como una aproximación al enfoque cuántico subyacente. Consulte la Tesis de Feynman: un nuevo enfoque de la teoría cuántica o una llamada a la acción , de Edwin F. Taylor .

Básicamente, todo se resume en pocas palabras en Richard P. Feynman, The Feynman Lectures on Physics (Addison–Wesley, Reading, MA, 1964), vol. II, cap. 19. (Creo, por favor corrígeme si me equivoco aquí). La idea fundamental es que la integral de acción define la amplitud mecánica cuántica para la posición de la partícula, y la amplitud es estable a los efectos de interferencia (--> tiene una probabilidad de ocurrencia distinta de cero) solo en los extremos o puntos de silla de la integral de acción. La partícula realmente explora probabilísticamente todos los caminos alternativos.

Es probable que desee leer Lectures on Physics de Feynman de todos modos, por lo que es mejor que comience ahora. :-)

Las Lectures on Physics de Feynman son buenas, pero creo que es mejor leerlas después para haber aprendido correctamente el tema, a fin de proporcionar una visión nueva / adicional.
Pero, ¿por qué es válido el principio de acción mínima estacionaria en la teoría cuántica? Esa es una pregunta aún más difícil de responder. Explicar la mecánica newtoniana como "es un límite de la teoría cuántica" no es muy pedagógico.
Debido a la aproximación de la fase estacionaria.

Generalmente cuento la historia de que el principio de acción es otra forma de llegar a las mismas ecuaciones diferenciales, por lo que al nivel de la mecánica, las dos son equivalentes. Sin embargo, cuando se trata de la teoría cuántica de campos, la descripción en términos de integrales de trayectoria sobre la acción exponenciada es esencial al considerar los efectos instantónicos. Así que eventualmente uno encuentra que la formulación en términos de acciones es más fundamental y físicamente más sólida.

Pero aun así, las personas no tienen un "sentido" de la acción de la misma manera que tienen un sentido de la energía.

Como puede ver en la imagen a continuación, desea que la variación de la integral de acción sea mínima, por lo tanto d S d q debe ser 0 . De lo contrario, no está tomando el verdadero camino entre q t 1 y q t 2 pero un camino un poco más largo. Sin embargo, aun siguiendo d S = 0 , como sabes, podrías terminar con otro extremo.

texto alternativo

Siguiendo el enlace de jc, puede encontrar On a General Method on Dynamics , que probablemente responda su pregunta sobre el razonamiento de Hamilton. No lo he leído pero casi seguro que merece la pena.

Esto parece una respuesta tautológica ya que es precisamente el principio de Hamilton el que se usa para llegar a la imagen de arriba en primer lugar.
Tal vez le enseñaron el principio de Hamilton y llegó a esa imagen como explicación, pero la imagen es perfectamente general. Describe la variación de una función con puntos finales fijos.

Recordemos que las ecuaciones de movimiento con condiciones iniciales q ( 0 ) , ( d q / d t ) ( 0 ) se propusieron primero y el principio de acción mínima se formuló después, como una secuencia. Aunque es hermoso y elegante desde el punto de vista matemático, el principio de acción mínima utiliza alguna condición futura de "límite". q ( t 2 ) , que se desconoce físicamente. No existe un principio de acción mínima que opere únicamente con las condiciones iniciales.

Además, se da a entender que las ecuaciones tienen soluciones físicas. Esto es así en la Mecánica Clásica pero es erróneo en la Electrodinámica Clásica. Entonces, incluso derivadas de un "principio" formalmente correcto, las ecuaciones pueden ser incorrectas a nivel físico y matemático. A este respecto, formular las ecuaciones físicas correctas es una tarea más fundamental para los físicos que confiar en algún "principio" de obtención de ecuaciones "automáticamente". Somos los físicos los responsables de formular correctamente las ecuaciones.

En CED, QED y QFT, uno tiene que "reparar sobre la marcha" las soluciones incorrectas solo porque la física se adivinó e inicialmente se implementó incorrectamente.

PS Me gustaría mostrar cómo en realidad el sistema "elige" su trayectoria: si en t = 0 la partícula tiene un momento pag ( t ) , entonces en la próxima vez t + d t tiene el impulso pag ( t ) + F ( t ) d t . Este incremento es bastante local en el tiempo, está determinado por el valor actual de la fuerza F ( t ) por lo que ninguna condición futura de "límite" puede determinarlo. La trayectoria no se "elige" entre las virtuales; es "dibujado" por los valores instantáneos de fuerza, coordenadas y velocidad.

Me gusta pensar que ambas opciones son meramente modelos matemáticos, por lo que ninguna es más real. Ni el sistema elige su trayectoria ni el futuro determina el camino de mínima acción. La no localidad de QM lleva a dudas similares.
Sorprendentemente, ¡ahora existe un principio de acción mínima que opera solo con las condiciones iniciales! prl.aps.org/abstract/PRL/v110/i17/e174301
Aquí hay una versión gratuita de arXiv . Sin leer el artículo en detalle, huele a formalismo keldysh clásico , cf. esto y esto Phys.SE publicaciones.
> "se da a entender que las ecuaciones tienen soluciones físicas. Esto es así en la Mecánica Clásica pero está mal en la Electrodinámica Clásica" La mecánica clásica también tiene soluciones no físicas. Con el sistema correcto y las condiciones iniciales, uno puede romper el determinismo (cúpula de Norton) y la entropía de Gibbs (esperada de la termodinámica) no decreciente en el tiempo en sistemas aislados. Parece que las soluciones físicas siempre son elegidas por los físicos, nuestras teorías siempre permiten soluciones no físicas.

En lugar de especificar la posición inicial y el momento tal como lo hemos hecho en el formalismo de Newton, reformulemos nuestra pregunta de la siguiente manera:

Si optamos por especificar las posiciones inicial y final: ¿Qué camino toma la partícula?

ingrese la descripción de la imagen aquí

Afirmemos que podemos recuperar el formalismo de Newton mediante el siguiente formalismo, el llamado formalismo lagrangiano o principio hamiltoniano.

A cada camino ilustrado en la figura anterior, le asignamos un número que llamamos la acción

S [ r ( t ) ] = t 1 t 2 d t ( 1 2 metro r ˙ 2 V ( r ) )

donde este integrando es la diferencia entre la energía cinética y la energía potencial.

Las principales afirmaciones de Hamilton : El verdadero camino tomado por la partícula es un extremo de S.

Prueba:

1. Cambia ligeramente la ruta:

r ( t ) r ( t ) + d r ( t )

2.Mantenga fijos los puntos finales de la ruta:

d r ( t 1 ) = d r ( t 2 ) = 0

3. Toma la variación de la acción. S :

ingrese la descripción de la imagen aquí

finalmente, obtendrás

d S = t 1 t 2 [ metro r ¨ V ] d r

La condición de que el camino con el que comenzamos sea un extremo de la acción es

d S = 0

que debe mantenerse para todos los cambios d r ( t ) que hacemos al camino. La única forma en que esto puede suceder es si la expresión en [ ] es cero Esto significa

metro r ¨ = V

Ahora reconocemos esto como ecuaciones de newton . Exigir que la acción se extremice es equivalente a exigir que el camino obedezca las ecuaciones de Newton.

Para obtener más detalles, puede leer esta conferencia en pdf.

Espero eso ayude.

Si vemos una partícula restringida para moverse en una esfera, llegamos a caminos uno es un máximo o un mínimo. Siento que una partícula sigue el camino de mínima acción, pero la ecuación matemática δS=0 nos da una respuesta ambigua, pero cierta parte de esta respuesta contiene un camino de mínima acción. Puedes ver Arfken y Weber.

Es posible en la física clásica derivar las ecuaciones de Euler-Lagrange del principio de D'Alembert, sin ninguna referencia a la noción de acción. Vienen de las leyes de Newton con la suposición adicional de que las fuerzas son conservativas. En este caso hay un Lagrangiano, y la ecuación de movimiento (EOM) es la ecuación de Euler-Lagrange.

Supongamos que una función q(t) es una solución para el MOE en un cierto intervalo. q se puede expandir como una serie de Taylor, es decir, una serie de potencias: q ( t ) = j a j t j .

La acción es: S ( L ) = t 1 t 2 L d t donde L es el Lagrangiano que corresponde a la EOM. porque la integral esta en t , y estamos tomando la derivada con respecto a los coeficientes a j , puede ir dentro de la integral. Para cada a j .

S a j = t 1 t 2 L a j d t

L es una función de q y q ˙ , por lo que aplicando la regla de la cadena:

L a j = L q q a j + L q ˙ q ˙ a j

Integrando este diferencial entre 2 instantes de tiempo:

t 1 t 2 L a j d t = t 1 t 2 L q q a j d t + t 1 t 2 L q ˙ q ˙ a j d t

El último término se puede separar usando integral por partes, usando eso diferenciando con respecto al tiempo: d ( q a j ) = q ˙ a j d t :

t 1 t 2 L q ˙ q ˙ a j d t = L q ˙ q a j | t 1 t 2 t 1 t 2 L q ˙ t q a j d t

Entonces:

t 1 t 2 L a j d t = t 1 t 2 L q q a j d t t 1 t 2 L q ˙ t q a j d t + L q ˙ q a j | t 1 t 2

Uniendo las integrales, obtenemos entre paréntesis la ecuación de Euler-Lagrange, ¡esa es la propia MOE! Si q es solución por hipótesis, esta integral debe ser cero.

t 1 t 2 L a j d t = t 1 t 2 ( L q L q ˙ t ) q a j d t + L q ˙ q a j | t 1 t 2

Para el último término, la integral de segundo orden necesita 2 condiciones de contorno. Si q ( t 1 ) y q ( t 2 ) son conocidos, son fijos y q a j | t 1 = q a j | t 2 = 0 este término se desvanece.

Ahora, llegamos a la conclusión de que la derivada de la acción con respecto a todos los coeficientes debe ser cero en el intervalo, lo que es lo mismo que decir que la acción debe ser estacionaria.

Sí, derivar de clásico. La cuestión es que el trabajo virtual de d'Alembert es una forma no transparente de enunciar el teorema del trabajo y la energía. Es por eso que los libros de hoy presentan el teorema trabajo-energía, no el de d'Alembert. Tenemos que el teorema trabajo-energía es otra forma de enunciar F = metro a Demostración: teorema trabajo-energía:
s 0 s F   d s = 1 2 metro v 2 1 2 metro v 0 2
hacer que la velocidad inicial sea cero. Derivar con respecto a la posición:
d ( 1 2 metro v 2 ) d s = metro a
La acción estacionaria de Hamilton funciona porque es matemáticamente equivalente al teorema del trabajo y la energía.
@Cleonis, el principio de D'Alembert no es una declaración sobre el trabajo o la energía como en el teorema de trabajo y energía. Es una reformulación de las ecuaciones de movimiento newtonianas. Esto es equivalente a esas ecuaciones y, por lo tanto, el principio es más poderoso que el teorema del trabajo y la energía.
@JánLalinský El teorema del trabajo y la energía es matemáticamente equivalente a F = metro a Toma la derivada con respecto a la posición y recuperas F = metro a
F   d s d s = F
y
d ( 1 2 metro v 2 ) d s = 1 2 metro ( 2 v d v d s ) = metro d s d t d v d s = metro d v d t = metro a
@Cleonis, su cálculo da la misma ecuación que las leyes de Newton solo para el movimiento lineal. Si la partícula se mueve a lo largo de una trayectoria curva, d mi k / d s solo nos da la componente tangencial de la fuerza (en la dirección de la velocidad), no la fuerza total. Por ejemplo, no obtendrá el componente centrípeto de la fuerza durante un movimiento circular simple de esta manera. El teorema trabajo-energía puede darnos solo la parte de la fuerza que cambia la energía cinética, no puede darnos el componente normal, por lo tanto, no puede darnos todas las ecuaciones que hacen las leyes de Newton.
@JánLalinský Cuando el vector de fuerza y ​​el vector de velocidad no están alineados, usa el producto vectorial. Consulte el siguiente PDF de OpenCoursesWare: Módulo 28: el problema de Kepler: mecánica planetaria El movimiento se describe en términos de coordenadas polares. A partir de la sección 28.6, el tratamiento demuestra que la evaluación de la energía se puede utilizar para resolver el problema de Kepler. Solución general: incluye órbitas excéntricas.
Los comentarios de @JánLalinský stackexchange no son para discusiones prolongadas. Le recomiendo encarecidamente que envíe su punto como una pregunta de intercambio de pila . Envíe su punto de vista a la comunidad de stackexchange. Enlace a ese PDF de openCourseWare y pregunte: "Creo que el teorema de trabajo-energía solo puede abordar casos con movimiento lineal. Sin embargo, en este documento OCW de la sección 28.6 sobre el problema de Kepler se resuelve mediante la evaluación de energía. ¿Qué da?"
@Cleonis No puedo acceder al documento vinculado. Estoy de acuerdo que este no es el lugar adecuado para discutir esto. Solo quería señalarle que sus afirmaciones no son estándar y lo más probable es que sean incorrectas. Buena suerte.